Questions tagged [real-analysis]

For questions about real analysis, such as limits, convergence of sequences, properties of the real numbers, the least upper bound property, and related analysis topics such as continuity, differentiation, and integration.

Filter by
Sorted by
Tagged with
2 votes
2 answers
71 views

Is Lebesgue density independent of the choice of neighborhoods?

Let $E$ be a Borel subset of $\mathbb R^d$ and $m$ be the Lebesgue measure on $\mathbb R^d$ and by definition of density (Folland real analysis 2nd edition ,Exercise 25 in page 100): $$D_E(x):= \lim_{...
No One's user avatar
  • 7,939
0 votes
1 answer
96 views

Proving a function continuous at $0$.

Define for $-\pi<\arg z<\pi$ $$f(z)=\frac{1}{1-\sqrt{z}}$$ and define $$f(0)=1$$ Question Prove that $f(z)$ is continuous at $0$. My try- Define $$g(z)=\sqrt{z}$$ Define $g(0)=0$. Let, $\epsilon&...
Maths's user avatar
  • 19
2 votes
1 answer
101 views

Asymptotic Estimation of a function ${\int_{0}^{x} \frac{\sin^2 t}{t^2} dt}$

I'm reading a book "Theory of Approximation of Functions of a Real Variable" by A.F Timan On the page 14 there is an asymptotic estimation:"... It is an integral function of the first ...
Metso's user avatar
  • 708
2 votes
0 answers
69 views

Confusion regarding solution manual's entry to Question 3b Chapter 8 of Spivak's Calculus

Question 3b from Chapter 8 of Spivak's Calculus reads as: The proof of the Theorem 7-1 depended upon considering $A = \{x: a \leq x \leq b \text{ and } f \text{ is negative on } [a,x]\}$. Give ...
S.C.'s user avatar
  • 4,984
0 votes
1 answer
21 views

Bounds of double integrals determined by a minimum

I'm studying for exams and in the previous years there has been a question on a double integral where the domain has the condition : $ |y| <= min(1,2-x) $ My problem is that I don't understand how ...
Facari's user avatar
  • 5
1 vote
2 answers
365 views

The simplest ODE $x' = x$

An ODE $x'(t) = x(t)$ is one of the simplest equation. If $x(t) \in C^1(\mathbb{R})$ is a solution of this ODE, we get $x(t) = C \exp(t)$, where $C$ is constant. This is very elementary fact in ODE ...
heppoko_taroh's user avatar
0 votes
1 answer
276 views

Finding Limit vs Proving Limit of *something*.

I get confused everytime between finding a limit and proving that something is a limit. If I calculate the limit of a function $f(x)$ at $a$ and say, I get $\lim\limits_{x \to a} f(x) =L$ where $a\in [...
William's user avatar
  • 4,893
2 votes
4 answers
2k views

Prove that $|x|<\epsilon$ for every $\epsilon$ greater than $0$ if and only if $x$ is equal to zero.

Prove that $|x|<\epsilon$ for every $\epsilon$ greater than $0$ if and only if $x$ is equal to zero. My Attempt: Assume $x$ to be a non-zero number, say $x=2$. Clearly there is a contradiction here ...
Maverick's user avatar
  • 9,220
0 votes
0 answers
42 views

9.8 Mathematical Analysis by Apostol

Can someone please explain to me the 2nd point in question below? In point 2 what does the condition $|f_k(x)-f(x)|<\delta$ means, is it true for all $k$ or does this $k$ depends on $x$?
Arun's user avatar
  • 91
6 votes
1 answer
238 views

If $a_n=\sqrt{1+\sqrt{2+\cdots\sqrt{n}}}$ and $\lim\limits_{n\to\infty}a_n=\ell$, prove $\lim_{n\to\infty}[(\ell-a_n)^{1/n}n^{1/2}]=\frac{\sqrt e}2$

$$a_n=\sqrt{1+\sqrt{2+\cdots\sqrt{n}}}$$ We can prove that $\{a_n\}$ is convergent (using mathematical induction, $\sqrt {k+\sqrt{k+1+\cdots\sqrt{n}}}\leq k-1, for \ k\geq3$). If $$ \lim\limits_{n\to\...
William Leynoid's user avatar
0 votes
2 answers
744 views

If the flux of a vector field $F$ is zero for ANY surface $S$ then is $F$ zero?

If $\vec F$ is a vector field defined in a open (contractible) set $U$ of $\Bbb R^3$ such that $$ \int_S\vec F\cdot\hat ndS=0 $$ for ANY surface $S$ then in particular the last equality holds for the ...
Antonio Maria Di Mauro's user avatar
0 votes
1 answer
51 views

Rigorous proof of existence of a unique function whose arc-length is minimal.

Let $a,b, \alpha, \beta \in \Bbb R$, $F = \{ f \in C^1([a,b]) \mid f(a)=\alpha, f(b)=\beta\}$ and $L : F \to \Bbb R^+, f \mapsto \int_{a}^b \sqrt{1+f'(x)^2} dx$. Prove that $L$ has a unique minimum. ...
Michelle's user avatar
  • 1,784
2 votes
0 answers
54 views

Show that $\lim\limits_{t \to 0}\frac{Det(A+tB)-Det(A)}{t} = Det(A)\cdot Trace(A^{-1}B)$

Show that $\lim\limits_{t \to 0}\frac{Det(A+tB)-Det(A)}{t} = Det(A)\cdot Trace(A^{-1}B)$. Also show that $\langle \nabla Det(A),B \rangle=Det(A)\cdot Trace(A^{-1}B)$. I know that $det(tI_{n\times n}+C)...
Arius's user avatar
  • 67
0 votes
1 answer
53 views

Integral remainder from Taylor's formula

The wikipedia version of the integral remainder from Taylor's theorem says (if I want an order 2 expansion)$$f(y)=f(x)+f'(x)(y-x)+{1\over2}f''(x)(y-x)^2+{1\over2}\int\limits_x^y(y-z)^2f'''(z)dz$$ But ...
H. Walter's user avatar
  • 979
0 votes
2 answers
116 views

Can any $f:\mathbb N \to \mathbb N$ be differentiable at a point?

I know that any function on $\mathbb N$ is continuous but is any function differentiable on $\mathbb N$? For example if $f(x) = x^2, f: \mathbb N \to \mathbb N$ and let $c = 2$, can we claim the ...
mathcomp guy's user avatar
0 votes
0 answers
40 views

Let $f: \Bbb{R^n} \rightarrow \Bbb{R}$ be a function such that $\forall q \in \Bbb{Q}$, the sublevel set $\{x: f(x)<q\}$ is open...

This is a question in old qualification exam. Given the condition mentioned by the title, I am asked: a) Must $f$ be Borel-measurable? b) Must $f$ be continuous? First, I know that b) is true since ...
James C's user avatar
  • 1,338
0 votes
0 answers
270 views

Limit of the convolution of two functions as the product of the integral of one and the limit of the other

Let $\lambda, h : \mathbb{R} \to \mathbb{R}$ and suppose $\int_{-\infty}^\infty h(t)\,dt$ exists and $\lim_{t \to \infty}\lambda(t)$ exists. I want to show that the limit of their convolution equals ...
Alojaco's user avatar
  • 43
0 votes
0 answers
97 views

Does differentiating a differentiable function give a differentiable function?

I want to know that if a differentiable function is differentiated, it is still differentiable. I mean, let $f \ $ be a differentiable function, and $f' \ $ its derivative. Is $f' \ $ always also a ...
Sigrid's user avatar
  • 71
0 votes
2 answers
329 views

metric space without isolated points

Let $X$ be a compact metric space with no isolated points (so every point is a limit point). Show that $X$ is uncountable. I think I can show this by showing that $X$ has cardinality at least that of ...
Alfred's user avatar
  • 859
0 votes
1 answer
64 views

Inequality with a matrix norm

Let $T\in \mathcal L(\mathbb R^n, \mathbb R^m)$ and $A = [a_{ij}] \in \mathcal M_{m\times n}$ the associated matrix of this transformation. If $\mu := \max \{ |a_{ij}| : i =1, \dots, m; j = 1, \dots, ...
Keio203's user avatar
  • 551
3 votes
3 answers
664 views

connected and path components

Let $F = G\cup H$ where $G = \{(x,y) \in \mathbb{R}^2 : x=0, y = [0,1]\cap \mathbb{Q}\}$ and $H = \{(x,y) \in \mathbb{R}^2 : x > 0, y \in [0,1]\backslash \mathbb{Q}$. Find the connected components ...
Alfred's user avatar
  • 859
1 vote
0 answers
40 views

Perpendicular continuous function of $R^3$ unit vectors?

Let S be the set of all unit vectors in $\mathbb{R}^3$ Does there exist a continuous function $f$ from $S \to S$ such that $\forall v \in S: v \cdot f(v) = 0 $ ? If so, what is one such function? If ...
Andrew Tomazos's user avatar
0 votes
0 answers
35 views

Finding certain improper integrals

I need to find two functions, $f, g : (0,1] \rightarrow \mathbb{R} $, such that both integrals $\int_{0}^{1} f(x) dx $ and $\int_{0}^{1} g(x) dx $ are improper and exist but the product is an ...
Parinn's user avatar
  • 537
1 vote
1 answer
395 views

metric spaces with certain topological properties

Define for a field $\mathbb{F} = \mathbb{R}$ or $\mathbb{C}, \mathbb{F}^\omega := \{(x_n)_{n\geq 1} : x_i \in \mathbb{F}\,\forall i\}.$ Let $x=(x_n)_{n\geq 1}, y=(y_n)_{n\geq 1} \in \mathbb{R}^\omega$....
Fred Jefferson's user avatar
23 votes
1 answer
1k views

Delightful integral: $\int _0^{\frac{\pi }{2}}x\ln \left(\sin \left(x\right)\right)\ln ^2\left(\cos \left(x\right)\right)\:dx.$

A friend of mine proposed me this integral which I find to be very interesting. I managed to find with the help of software that: $$\int _0^{\frac{\pi }{2}}x\ln \left(\sin \left(x\right)\right)\ln ^2\...
Jorge Layja's user avatar
  • 1,111
1 vote
1 answer
73 views

Determine the convergence of following series

Determine the convergence of following series. $$ \sum_{n=1}^\infty a_n$$ where $$ a_n = \begin{cases} \bigg( \dfrac{2+3n}{5+6n} \bigg)^n, & \text{if $n$ is even} \\ 5 & \text{if $n$ is ...
ALMEra's user avatar
  • 357
3 votes
0 answers
95 views

Baby Rudin 1.20(b) Proof—Alternative Justification

This question is based on a previously asked one that I will quote in full here: I have a question about Rudin's proof of Theorem 1.20 (b) in his book Principles of Mathematical Analysis. Theorem 1....
Joe's user avatar
  • 19.7k
0 votes
0 answers
65 views

How does the standard topology on $R$ induce a structure on $R$?

I'm having a hard time seeing how does the standard topology on $R$ induces the structure on the set of real numbers such that it give us the real number line! The definition of a topology on a set ...
I0_0I's user avatar
  • 569
0 votes
3 answers
341 views

Definition of Sequence Convergence Doesn't Make Sense

In real analysis, the convergence of a sequence is defined as follows: The statement that $|a_n - a|<\varepsilon$ means that the range is $[a-\varepsilon, a+\varepsilon]$, where $\varepsilon>0$....
Bitskit's user avatar
  • 11
0 votes
0 answers
63 views

Given $ f: [a, b] \to \mathbb R $ and $ n \in \mathbb N $, assume $ h = \frac {b-a}{}\cdots$

Given $ f: [a, b] \to \mathbb R $ and $ n \in \mathbb N $, assume $ h = \frac {b-a} {n} $ and the average of the numbers $ f ( a + h ), \ldots, f (a + nh) = f (b) $ given by $$ M (f; n) = \frac {1}{n} ...
Andre's user avatar
  • 69
2 votes
0 answers
34 views

if $f$ is continuous over some interval $[z_1,z_2]$, then $f(bx+c)$ is continuous over the adjusted interval $[\frac{z_1-c}{b}, \frac{z_2-c}{b}]$

I want to prove that if $f$ is continuous over some interval $[z_1,z_2]$, then any horizontally translated and horizontally "compressed/stretched" version of $f$ (i.e. a function of the form ...
S.C.'s user avatar
  • 4,984
1 vote
1 answer
408 views

Conjugate Gradient and Krylov Subspace Method

In HF, on each iteration the CG algorithm is used to approximately compute $$ \mathbf{d}=-(\mathbf{H}+\lambda \mathbf{I})^{-1} \mathbf{g} $$ where $\mathrm{d}$ is the step direction, and $\mathrm{g}$ ...
maths student's user avatar
1 vote
2 answers
88 views

$f(X)\subset \{0\}\cup f(\text{supp}(f))$ vs $f(X)\subset f(\text{supp}(f))$

Suppose $f$ has compact support and is continuous (say we are in the simplest case of $f:\mathbb R\rightarrow \mathbb R$). I was reading this answer, and can't understand why there needs to be a $\{0\}...
H. Walter's user avatar
  • 979
1 vote
0 answers
50 views

Bounding the total variation of an Absolute Continuous function

Theorem: Suppose $f \in AC([a,b])$. Prove that $$ V_a^b(f) \leq ||(f')^+||_{L^1([a,b])} + ||(f')^-||_{L^1([a,b])} $$ Proof (my attempt): If $f \in AC([a,b])$ then calculus formula holds and we can ...
AlePalu's user avatar
  • 58
0 votes
1 answer
57 views

Proof that $x^2$ is continuous by $x = x_0 +h$ definition

Let's first sketch the proof: $$|f(x_0 + h) - f(x_0)| = |(x_0 +h)^2 - x_0^2| = |h^2 + 2x_0h| \leq|h||h + 2x_0|...\text{Now let's say that |h|$< \delta$ $...$}\\ |h||h + 2x_0|< |h||1 + 2x_0| <...
VLC's user avatar
  • 2,527
3 votes
3 answers
125 views

Let $a_n$ be a sequence of nonnegative numbers such that $a_n \rightarrow b$. Then, $\lim_{n \rightarrow \infty} \frac{(a_{n+1})^{n+1}}{(a_n)^n}$ =?

As the title says, I would like to evaluate the following limit given that $a_n \rightarrow b < \infty$: $\lim_{n \rightarrow \infty} \frac{(a_{n+1})^{n+1}}{(a_n)^n}$ (I am assuming that the limit ...
James C's user avatar
  • 1,338
1 vote
0 answers
161 views

Approximation for log of convex combination

Thinking about the approximation of $$ \ln \sum_{i=1}^{N}\theta_{i}x_{i}, $$ where $\{\theta_i\}$ are positive constant that satisfies $\sum_{i=1}^{N}\theta_i= 1$ (i.e., weights) and $\{x_{i}\}$ are ...
Kohei's user avatar
  • 11
2 votes
2 answers
3k views

Different definition for L smooth function.

At optimization class, professor gave the definition of L smooth function by $f:\mathbf{R^n} \rightarrow \mathbf{R} $ is L smooth if all the eigenvalue for $\nabla^{2} f $ is smaller than L where $\...
Archer914's user avatar
0 votes
0 answers
55 views

Continuity of finite intersection of continuous correspondence

I was solving a problem which requires me to show the continuity of a set-valued map. Suppose $\Bigl(T(x_i)\Bigr)_{i=1}^{n} \subset A$, where the set-valued map $T$ defined as $\mathbb{X} \ni x \...
souvikd's user avatar
  • 41
4 votes
2 answers
1k views

If $f$ is strictly increasing on $[a,b]$ then $f$ is continuous at some point in the interval.

Edit: Proof version 2.0 Thanks to all for your insight and suggestions. Theorem: If a function $f$ is strictly increasing on the closed interval $[a,b]$, then $f$ is continuous at some point in the ...
Ben's user avatar
  • 1,585
2 votes
0 answers
67 views

Attempt at prove that rank theorem implies that the image of this function with constant rank is a manifold

Let $f:U\rightarrow U$, $U\subseteq\mathbb{R^m}$ open, such that $f\circ f = f$ and let $M=f(U)$. I showed that there exists neighborhood $V\subseteq U$ of M such that $f$ has constant rank on $V$, ...
Fernando Nazario's user avatar
1 vote
2 answers
333 views

Inverse Function Theorem and finding local inverses

$f(x,y) = (x+y,x^2+y)$ for this function, the question is Write down a $C^1$ local inverse around (0, 0). I wonder how can I find the points f has a local inverse? I am trying to use Lagrange ...
epsilon's user avatar
  • 43
1 vote
0 answers
44 views

If $q(t)\underset{t\to +\infty}{\to}1$ then number of zeros of solution of $x^{''}+q(t)x=0$ verify $N(t)\underset{t\to +\infty}{\sim}\frac{t}{\pi}$

Let $q:\mathbb{R}^+\to \mathbb{R}$ continuous function such that $q(t)\underset{t\to +\infty}{\to}1$. We consider $\phi$ solution of differential equation : $x^{''}+q(t)x=0$ and $N(t)$ is the number ...
Kim Kim's user avatar
  • 61
2 votes
1 answer
231 views

Equivalence of definitions of the Axiom of Continuity in Economics

Statement $1$: If $(a_n, b_n)_{n \geq 1}^{\infty}$ is a sequence in $X \times X$ satisfying $a_n \succsim b_n$ and $\displaystyle\lim_{n \to \infty} (a_n, b_n) = (a,b)$, then $a \succsim b$. Statement ...
dictatemetokcus's user avatar
1 vote
1 answer
74 views

Finding the interior and boundary of a subset of $(l^1,||\cdot||_{1})$

Consider the folllowing metric space: $X = (l^1,||\cdot||_{1})$. Recall $$l^1 = \big\{\{x_k\}_{k=1}^{\infty}: \sum_{k=1}^{\infty}|x_k| < \infty\big\}.$$ Let $Y \subset X$ be the set of sequences $\{...
user avatar
0 votes
1 answer
286 views

Uniform and pointwise convergence of $x^n$ on $[0,1]$

Show that the sequence of functions $f_n(x) = x^n$ defined on $[0, 1]$ converges pointwise but not uniformly. Now suppose $g \in C([0, 1])$ such that $g(1) = 0$. Show that the sequence $f_ng$ ...
epsilon's user avatar
  • 43
1 vote
1 answer
124 views

Inequalities for limit supremum

We know that for any bounded sequences $(s_n)$ and $(t_n)$ the inequality holds $\limsup (s_n+t_n) \leq \limsup (s_n) + \limsup (t_n)$. I understand how to prove this inequality, but why doesn't this ...
Some_Math_Nerd's user avatar
2 votes
1 answer
513 views

Question regarding the fourier series, the L2 space of square integrable functions and convergence.

I have been self learning the fourier series and if anyone can help me with my questions that will be super great! Firstly, why is the fourier series defined on the L2 space and not on other spaces? ...
woowz's user avatar
  • 153
0 votes
0 answers
38 views

Global minimum and local minimum question and proof of $x^2$ local minimum at $x = 0$

We defined $f : [a,b] \rightarrow \mathbb{R}$ has a global maximum in $x_0 \in [a,b]$ on $[a,b]$ if $x \neq 0 \implies f(x) < f(x_0)$ Then we also defined local maximum: $f : [a,b] \rightarrow \...
VLC's user avatar
  • 2,527
2 votes
1 answer
309 views

Proof verification of theorem regarding $m$-tail of a sequence

Prove that a sequence $X=(x_n)$ converges iff its $m$-tail $X_m$($m\in \mathbb N$) converges. For that case, $lim X=lim X_m$. Proof:If $X$ converges to $x$, then for a given $\varepsilon>0$, $\...
Edmund Blackadder's user avatar

1
458 459
460
461 462
2913